LSAT and Law School Admissions Forum

Get expert LSAT preparation and law school admissions advice from PowerScore Test Preparation.

User avatar
 Dave Killoran
PowerScore Staff
  • PowerScore Staff
  • Posts: 5852
  • Joined: Mar 25, 2011
|
#43085
Complete Question Explanation
(The complete setup for this game can be found here: lsat/viewtopic.php?f=1250&t=16333)

The correct answer choice is (B).


If M is not included in the tour, then the group of cities consists of H, T, J, and either O or S. As discussed in the setup (under scenarios 2 and 3), this forces J to be third, and then one of H or T is first or second, and the other is fourth (in order to conform to the first rule):

  • ___ ..... ___ ..... _J_ ..... _T/H_
     1 .....    2 .....    3 .....    4
The first two visits are occupied by the remainder of H/T and O or S, in some order.


Answer choice (A): While this answer could be true, it does not have to be true, and thus this answer choice is incorrect.

Answer choice (B): This is the correct answer choice.

Answer choice (C): While this answer could be true, it does not have to be true, and thus this answer choice is incorrect.

Answer choice (D): While this answer can never occur, and thus this answer choice is incorrect.

Answer choice (E): While this answer could be true, it does not have to be true, and thus this answer choice is incorrect.

Get the most out of your LSAT Prep Plus subscription.

Analyze and track your performance with our Testing and Analytics Package.